(27 intermediate revisions by 2 users not shown)
Line 1: Line 1:
= [[ECE PhD Qualifying Exams|ECE Ph.D. Qualifying Exam]] in "Communication, Networks, Signal, and Image Processing" (CS)  =
+
[[Category:ECE]]
 +
[[Category:QE]]
 +
[[Category:CNSIP]]
 +
[[Category:problem solving]]
 +
[[Category:image processing]]
  
= [[ECE-QE_CS5-2011|Question 1, August 2011]], Part 1 =
+
<center>
 +
<font size= 4>
 +
[[ECE_PhD_Qualifying_Exams|ECE Ph.D. Qualifying Exam]]
 +
</font size>
  
:[[ECE-QE_CS5-2011_solusion-1|Part 1]],[[ECE-QE CS1-2011 solusion-2|2]]]
+
<font size= 4>
 +
Communication, Networking, Signal and Image Processing (CS)
 +
 
 +
Question 5: Image Processing
 +
</font size>
 +
 
 +
August 2011
 +
</center>
 +
----
 +
----
 +
=Part 2=
 +
Jump to [[ECE-QE_CS5-2011_solusion-1|Part 1]],[[ECE-QE CS1-2011 solusion-2|2]]
  
 
----
 
----
Line 39: Line 57:
  
 
<math>
 
<math>
P_{\theta}(\rho)  = \int_{-\infty}^{+\infty}{\delta(r cos\theta - z sin\theta, r sin\theta + z cos \theta) dz}
+
p_{\theta}(r)  = \int_{-\infty}^{+\infty}{\delta(r cos\theta - z sin\theta, r sin\theta + z cos \theta) dz}
 
</math>
 
</math>
  
Line 45: Line 63:
 
<math>
 
<math>
 
= z \text{ when } \left\{\begin{matrix}
 
= z \text{ when } \left\{\begin{matrix}
r cos\theta - z sin\theta = 0  
+
r cos \theta - z sin \theta = 0
\\
+
\\  
 
r sin \theta + z cos \theta = 0
 
r sin \theta + z cos \theta = 0
 
\end{matrix}\right.
 
\end{matrix}\right.
 
</math>
 
</math>
 +
  
 
<math>
 
<math>
 
= \frac{r cos\theta}{sin \theta}, \theta > 0
 
= \frac{r cos\theta}{sin \theta}, \theta > 0
 +
</math>
 +
 +
<math>{\color{red}
 +
\text{This answer is incorrect. The correct answer is as following:}}
 +
</math>
 +
 +
<math>{\color{green}
 +
\text{Recall:}}
 +
</math>
 +
 +
 +
<math>{\color{green}
 +
\text{i) } \int_{-\infty}^{+\infty}{f(g(t)) \delta (t) dt} = f(g(t=0)) \int_{-\infty}^{+\infty}{\delta (t) dt}
 +
}</math>
 +
 +
 +
<math>{\color{green}
 +
\text{ii) } \int_{-\infty}^{+\infty}{\delta (\alpha t) dt} = \int_{-\infty}^{+\infty}{\delta (u) \frac{du}{|\alpha|}} = \frac{1}{|\alpha|}
 +
}</math>
 +
 +
 +
<math>{\color{green}
 +
\text{iii) } \delta() \text{ function is separable: } \delta(x,y) = \delta(x) \cdot \delta(y)
 +
}</math>
 +
 +
 +
<font face="serif"><span style="font-size: 19px;"><math>\color{green}
 +
\text{ Define } u = r cos\theta - z sin\theta
 +
</math></span></font>
 +
 +
 +
<math>\color{green}
 +
\Rightarrow dz = \frac{du}{|sin\theta|}
 +
</math>
 +
 +
 +
<font face="serif"><span style="font-size: 19px;"><math>\color{green}
 +
\text{ Now }
 +
</math></span></font>
 +
 +
 +
<math>\color{green}
 +
p_{\theta}(r)  = \int_{-\infty}^{+\infty}{\delta(r cos\theta - z sin\theta, r sin\theta + z cos \theta) dz}
 +
</math>
 +
 +
 +
<math>\color{green}
 +
p_{\theta}(r)  = \int_{-\infty}^{+\infty}{\delta(g(u)) \delta(u) \frac{du}{|sin\theta|}} = \frac{\delta(u=0)}{|sin\theta|}
 +
</math>
 +
 +
 +
<math>\color{green}
 +
= \frac{\delta(\frac{r}{sin\theta})}{|sin\theta|} = \frac{|sin\theta|}{|sin\theta|} \delta(r) = \delta(r)
 
</math>
 
</math>
  
Line 62: Line 134:
  
 
<math>
 
<math>
P_{\theta}(\rho)  = \int_{-\infty}^{+\infty}{\delta(r cos\theta - z sin\theta, r sin\theta + z cos \theta) dz}
+
p_{\theta}(r)  = \int_{-\infty}^{+\infty}{\delta(r cos\theta - z sin\theta, r sin\theta + z cos \theta) dz}
 
</math>
 
</math>
 +
  
 
<font face="serif"><span style="font-size: 19px;"><math>
 
<font face="serif"><span style="font-size: 19px;"><math>
 
= \delta(r)
 
= \delta(r)
 
</math></span></font>
 
</math></span></font>
 +
 +
 +
<font face="serif"><span style="font-size: 19px;"><math>{\color{green}
 +
\text{Here, the student uses the intuitive solution: in this case the answer does not depend on } \theta \text{, since the image just contains a peak at origin. }
 +
}</math></span></font>
  
 
----
 
----
Line 77: Line 155:
  
 
<math>
 
<math>
P_{\theta}(\rho)  = \int_{-\infty}^{+\infty}{\delta(r cos\theta - z sin\theta - 1, r sin\theta + z cos \theta - 1) dz}
+
p_{\theta}(r)  = \int_{-\infty}^{+\infty}{\delta(r cos\theta - z sin\theta - 1, r sin\theta + z cos \theta - 1) dz}
 
</math>
 
</math>
 
  
 
<math>
 
<math>
 
= z \text{ when } \left\{\begin{matrix}
 
= z \text{ when } \left\{\begin{matrix}
r cos\theta - z sin\theta = 1  
+
r cos \theta - z sin \theta = 1
\\
+
\\  
 
r sin \theta + z cos \theta = 1
 
r sin \theta + z cos \theta = 1
 
\end{matrix}\right.
 
\end{matrix}\right.
 
</math>
 
</math>
 +
  
 
<math>
 
<math>
Line 93: Line 171:
 
</math>
 
</math>
  
 +
<math>{\color{red}
 +
\text{This answer is incorrect. The correct answer is as following:}}
 +
</math>
 +
 +
<font face="serif"><span style="font-size: 19px;"><math>\color{green}
 +
\text{ Similar to the solution 1 to part a) we define u: } u = r cos\theta - z sin\theta - 1
 +
</math></span></font>
 +
 +
 +
<font face="serif"><span style="font-size: 19px;"><math>\color{green}
 +
\text{ Following the same logic as in part a) we obtain the final answer:}
 +
</math></span></font>
 +
 +
 +
<math>\color{green}
 +
p_{\theta}(r)  = \delta(r - (cos\theta + sin \theta)) = \delta(r - \sqrt{2} cos (\theta - \frac{\pi}{4}))
 +
</math>
  
 
----
 
----
Line 103: Line 198:
 
\tilde{p}_\theta(r) = p_{\theta}(r - \sqrt{1+1} cos(\theta - tan^{-1}(\frac{1}{1})))
 
\tilde{p}_\theta(r) = p_{\theta}(r - \sqrt{1+1} cos(\theta - tan^{-1}(\frac{1}{1})))
 
</math>
 
</math>
 +
  
 
<math>
 
<math>
 
= p_\theta(r - \sqrt{2} cos(\theta - \frac{\pi}{4}))
 
= p_\theta(r - \sqrt{2} cos(\theta - \frac{\pi}{4}))
 
</math>
 
</math>
 +
  
 
<math>
 
<math>
 
= \delta(r - \sqrt{2} cos(\theta - \frac{\pi}{4}))
 
= \delta(r - \sqrt{2} cos(\theta - \frac{\pi}{4}))
 
</math>
 
</math>
 +
 +
 +
<font face="serif"><span style="font-size: 19px;"><math>{\color{green}
 +
\text{Again, the student uses the intuitive solution: in this case the answer does depend on } \theta \text{, since the peak is shifted from the origin to the point } (1,1).
 +
}</math></span></font>
  
 
----
 
----
Line 123: Line 225:
  
 
<math>
 
<math>
P_{\theta}(\rho)  = \int_{-\infty}^{+\infty}{rect(\sqrt{(r cos\theta - z sin\theta)^2, (r sin\theta + z cos \theta)^2)} dz}
+
p_{\theta}(r)  = \int_{-\infty}^{+\infty}{rect(\sqrt{(r cos\theta - z sin\theta)^2 + (r sin\theta + z cos \theta)^2)} dz}
 
</math>
 
</math>
 +
 +
 +
<math>\color{green}
 +
\text{Recall should be added:}
 +
</math>
 +
 +
<math>\color{green}
 +
rect(t) = \left\{\begin{matrix}
 +
1, for |t|\leq \frac{1}{2}
 +
\\
 +
0, otherwise
 +
\end{matrix}\right.
 +
</math>
 +
 +
 +
<font face="serif"><span style="font-size: 19px;"><math>\color{green}
 +
\text{therefore: }
 +
</math></span></font>
  
  
 
<math>
 
<math>
= \int_{-\sqrt{\frac{1}{4} - r^2}}^{\sqrt{\frac{1}{4} - r^2}}{1 dz}
+
p_{\theta}(r)  = \int_{-\sqrt{\frac{1}{4} - r^2}}^{\sqrt{\frac{1}{4} - r^2}}{1 dz}
 
</math>
 
</math>
 +
  
 
<math>
 
<math>
 
= \left\{\begin{matrix}
 
= \left\{\begin{matrix}
&\sqrt{1 - 4r^2}, &\text{ if }|r| \leq \frac{1}{2}
+
\sqrt{1 - 4r^2}, &\text{ if }|r| \leq \frac{1}{2}
 
\\
 
\\
&0, &\text{ otherwise}
+
0, &\text{ otherwise}
 
\end{matrix}\right.
 
\end{matrix}\right.
 
</math>
 
</math>
 
  
 
----
 
----
Line 147: Line 267:
  
 
<math>
 
<math>
P_{\theta}(\rho)  = \int_{-\infty}^{+\infty}{f(r cos\theta - z sin\theta, r sin\theta + z cos \theta) dz}
+
p_{\theta}(r)  = \int_{-\infty}^{+\infty}{f(r cos\theta - z sin\theta, r sin\theta + z cos \theta) dz}
 
</math>
 
</math>
  
Line 155: Line 275:
 
</math>
 
</math>
  
<math>
+
 
\text{ else } P_{\theta}(\rho) = 0
+
<font face="serif"><span style="font-size: 19px;"><math>
</math>
+
\text{ else } p_{\theta}(r) = 0
 +
</math></span></font>
  
 
----
 
----
Line 168: Line 289:
  
 
<math>
 
<math>
P_{\theta}(\rho)  = \int_{-\infty}^{+\infty}{rect(\sqrt{(r cos\theta - z sin\theta - 1)^2, (r sin\theta + z cos \theta - 1)^2) dz}
+
p_{\theta}(r)  = \int_{-\infty}^{+\infty}{rect \left( \sqrt{(r cos\theta - z sin\theta - 1)^2 + (r sin\theta + z cos \theta - 1)^2} \right) dz}
 
</math>
 
</math>
  
Line 175: Line 296:
 
= \int_{-\sqrt{\frac{1}{4} - (r - (cos\theta + sin\theta))^2}}^{\sqrt{\frac{1}{4} - (r - (cos\theta + sin\theta))^2}}{1 dz}
 
= \int_{-\sqrt{\frac{1}{4} - (r - (cos\theta + sin\theta))^2}}^{\sqrt{\frac{1}{4} - (r - (cos\theta + sin\theta))^2}}{1 dz}
 
</math>
 
</math>
 +
  
 
<math>
 
<math>
 
= \left\{\begin{matrix}
 
= \left\{\begin{matrix}
&\sqrt{1 - 4(r - (cos\theta + sin\theta))^2}, &\text{ if }|r| \leq \frac{1}{2}
+
\sqrt{1 - 4(r - (cos\theta + sin\theta))^2}, &\text{ if }|r| \leq \frac{1}{2}
 +
\\
 +
0, &\text{ otherwise}
 +
\end{matrix}\right.
 +
</math>
 +
 
 +
 
 +
<math>\color{green}
 +
\text{ To make it more clear, the following form could be obtained:}
 +
</math>
 +
 
 +
<math>\color{green}
 +
= \left\{\begin{matrix}
 +
\sqrt{1 - 4(r - \sqrt{2} cos (\theta - \frac{\pi}{4}))^2}, &\text{ if }|r| \leq \frac{1}{2}
 
\\
 
\\
&0, &\text{ otherwise}
+
0, &\text{ otherwise}
 
\end{matrix}\right.
 
\end{matrix}\right.
 
</math>
 
</math>
Line 191: Line 326:
 
\tilde{p}_\theta(r) = p_{\theta}(r - \sqrt{1+1} cos(\theta - tan^{-1}(\frac{1}{1})))
 
\tilde{p}_\theta(r) = p_{\theta}(r - \sqrt{1+1} cos(\theta - tan^{-1}(\frac{1}{1})))
 
</math>
 
</math>
 +
  
 
<math>
 
<math>
 
= p_\theta(r - \sqrt{2} cos(\theta - \frac{\pi}{4}))
 
= p_\theta(r - \sqrt{2} cos(\theta - \frac{\pi}{4}))
 
</math>
 
</math>
 +
  
 
<math>
 
<math>
\text{ where } P_\theta(r) = \left\{\begin{matrix}
+
\text{ where } p_\theta(r) = \left\{\begin{matrix}
&\sqrt{1 - 4r^2}, &\text{ if }|r| \leq \frac{1}{2}
+
\sqrt{1 - 4r^2}, &\text{ if }|r| \leq \frac{1}{2}
 
\\
 
\\
&0, &\text{ else}
+
0, &\text{ else}
 
\end{matrix}\right.
 
\end{matrix}\right.
 
</math>
 
</math>
 +
 +
 +
<font face="serif"><span style="font-size: 19px;"><math>{\color{green}
 +
\text{Here, the student uses the results from solutions to part b and c.}
 +
}</math></span></font>
  
 
----
 
----
Line 211: Line 353:
 
<math>\color{blue}\text{Solution 1:}</math>  
 
<math>\color{blue}\text{Solution 1:}</math>  
  
<math>
+
<font face="serif"><span style="font-size: 19px;"><math>
 
1. \text{ Compute } \rho_{\theta}(r)
 
1. \text{ Compute } \rho_{\theta}(r)
</math>  
+
</math></span></font>  
  
<math>
+
 
 +
<font face="serif"><span style="font-size: 19px;"><math>
 
2. \text{ Compute FT of step 1.}
 
2. \text{ Compute FT of step 1.}
</math>  
+
</math></span></font>  
  
<math>
+
<font face="serif"><span style="font-size: 19px;"><math>
3. \text{ Multiply step 2 by the filter } H(\rho) = f_c \left [ rect(\frac{f}{2f_c}) - \Lambda(\frac{f}{f_c}) \right ]
+
3. \text{ Multiply step 2 by the filter } H(\rho) = |\rho| = f_c \left [ rect(\frac{f}{2f_c}) - \Lambda(\frac{f}{f_c}) \right ]
</math>
+
</math></span></font>  
+
 
<math>
+
<font face="serif"><span style="font-size: 19px;"><math>
 
4. \text{ Compute inverseFT of step 3.}
 
4. \text{ Compute inverseFT of step 3.}
</math>  
+
</math></span></font> 
  
 +
<math> \color{red}
 +
\text{ This answer is not complete. We need the final step:}
 +
</math>
 +
 +
 +
<font face="serif"><span style="font-size: 19px;"><math>\color{red}
 +
5. \text{ Back project } g_{\theta}(r) \text{ (obtained from step 4) and get: }
 +
</math></span></font>
 +
 +
<font face="serif"><span style="font-size: 19px;"><math>\color{red}
 +
f(x,y) = \int_{0}^{\pi}{g_\theta(xcos\theta + ysin\theta)d\theta}
 +
</math></span></font>
  
 
----
 
----
  
 
<math>\color{blue}\text{Solution 2:}</math><br>  
 
<math>\color{blue}\text{Solution 2:}</math><br>  
<math>
+
 
 +
<font face="serif"><span style="font-size: 19px;"><math>
 
1. \text{ Measure the projections } \rho_{\theta}(r) \text{ at various angles}
 
1. \text{ Measure the projections } \rho_{\theta}(r) \text{ at various angles}
</math>  
+
</math></span></font> 
  
<math>
+
<font face="serif"><span style="font-size: 19px;"><math>
2. \text{ Filter the projections } \rho_{\theta}(r) \text{ with } h(r) \text{, where } H(\rho) = |\rho| and get g_{\theta}(r)
+
2. \text{ Filter the projections } \rho_{\theta}(r) \text{ with } h(r) \text{, where } H(\rho) = |\rho| \text{ and get } g_{\theta}(r)
</math>  
+
</math></span></font> 
  
<math>
+
<font face="serif"><span style="font-size: 19px;"><math>
 
3. \text{ Back project } g_{\theta}(r) \text{ along } r = xcos\theta + ysin\theta \text{ and get }
 
3. \text{ Back project } g_{\theta}(r) \text{ along } r = xcos\theta + ysin\theta \text{ and get }
</math>
+
</math></span></font>  
 
   
 
   
<math>
+
<font face="serif"><span style="font-size: 19px;"><math>
 
f(x,y) = \int_{0}^{\pi}{g_\theta(xcos\theta + ysin\theta)d\theta}
 
f(x,y) = \int_{0}^{\pi}{g_\theta(xcos\theta + ysin\theta)d\theta}
 +
</math></span></font>
 +
 +
----
 +
=Solution 3:=
 +
 +
a)
 +
 +
<math>p_\theta(r)=\begin{cases}1, & \text{if } r=0\\0, & \text{otherwisr}\end{cases}=\delta(r)</math>.
 +
 +
[[Image:CW_QE_2011_CS5_2_a_sol3.png|200px]]
 +
 +
 +
b)
 +
 +
* If <math>0\leq\theta<\frac{\pi}{4}</math>, <math>d=\sqrt{2}\text{cos}(\frac{\pi}{4}-\theta)=\sqrt{2}[\text{cos}\frac{\pi}{4}\text{cos}\theta+\text{sin}\frac{\pi}{4}\text{sin}\theta]=\text{cos}\theta+\text{sin}\theta</math>.
 +
 +
&nbsp;&nbsp;&nbsp;&nbsp;&nbsp;&nbsp;&nbsp;&nbsp;[[Image:CW_QE_2011_CS5_2_b_1_sol3.png|300px]]
 +
 +
* If <math>\frac{\pi}{4}\leq \theta < \frac{\pi}{2}</math>, <math>d=\sqrt{2}\text{cos}(\theta-\frac{\pi}{4})=\sqrt{2}\text{cos}(\frac{\pi}{4}-\theta)=\text{cos}\theta+\text{sin}\theta</math>.
 +
 +
&nbsp;&nbsp;&nbsp;&nbsp;&nbsp;&nbsp;&nbsp;&nbsp;[[Image:CW_QE_2011_CS5_2_b_2_sol3.png|300px]]
 +
 +
* If <math>\frac{\pi}{2}\leq \theta < \frac{3\pi}{4}</math>, <math>d=\sqrt{2}\text{sin}(\frac{\pi}{4}-(\theta-\frac{\pi}{2}))=\sqrt{2}\text{sin}(\frac{3\pi}{4}-\theta)=\sqrt{2}[\text{sin}\frac{3\pi}{4}\text{cos}\theta-\text{cos}\frac{3\pi}{4}\text{sin}\theta]=\text{cos}\theta+\text{sin}\theta</math>.
 +
 +
&nbsp;&nbsp;&nbsp;&nbsp;&nbsp;&nbsp;&nbsp;&nbsp;[[Image:CW_QE_2011_CS5_2_b_3_sol3.png|300px]]
 +
 +
* If <math>\frac{3\pi}{4}\leq \theta < 2\pi</math>, <math>d=-\sqrt{2}\text{sin}(\theta-\frac{\pi}{2}-\frac{\pi}{4})=-\sqrt{2}\text{sin}(\theta-\frac{3\pi}{4})=-\sqrt{2}[\text{sin}\theta\text{cos}\frac{3\pi}{4}-\text{cos}\theta\text{sin}\frac{3\pi}{4}]=\text{cos}\theta+\text{sin}\theta</math>.
 +
 +
&nbsp;&nbsp;&nbsp;&nbsp;&nbsp;&nbsp;&nbsp;&nbsp;[[Image:CW_QE_2011_CS5_2_b_4_sol3.png|300px]]
 +
 +
<math>\Rightarrow p_\theta(r)=\delta(r-(\text{cos}\theta+\text{sin}\theta))</math> for all <math>\theta</math>.
 +
 +
 +
c)
 +
 +
Since <math>f(x,y)</math> is symmetric to <math>(0,0)</math>
 +
 +
<math>\Rightarrow p_\theta(r)=p_0(r)=\int_{-\sqrt{\frac{1}{4}-r^2}}^{\sqrt{\frac{1}{4}-r^2}}1\cdot dz=2\int_0^{\sqrt{\frac{1}{4}-r^2}}1\cdot dz\text{, for }\vert r\vert<\frac{1}{2}</math>
 +
 +
[[Image:CW_QE_2011_CS5_2_c_sol3.png|200px]]
 +
 +
<math>\Rightarrow p_\theta(r)=2\sqrt{\frac{1}{4}-r^2}\text{rect}(r)</math>
 +
 +
 +
d)
 +
 +
From part (b), we know that, if <math>f(x,y)</math> has <math>(1,1)</math> shift in <math>x-y</math> plane, the <math>p_\theta(r)</math> would have <math>(\text{cos}\theta+\text{sin}\theta)</math> shift in <math>r</math>-direction.
 +
 +
<math>\Rightarrow p_\theta(r)=2\sqrt{\frac{1}{4}-(r-(\text{cos}\theta+\text{sin}\theta))^2}\text{rect}(r-(\text{cos}\theta+\text{sin}\theta))</math>
 +
 +
 +
e)
 +
 +
* First, measure the forward projection <math>p_\theta(r)</math> for sufficient angles <math>\theta s</math>.
 +
* Second, filter the forward projection with <math>h(r): g_\theta(r)=h(r)\ast p_\theta(r)</math>, where <math>h(r)=\text{CTFT}^{-1}\left\{\vert\rho\vert\right\}</math>.
 +
* Finally, back project filtered projection to reconstruct <math>f(x,y)</math>. <math>f(x,y)=\int_0^\pi g_\theta(x\text{cos}\theta+y\text{sin}\theta)d\theta</math>.
 +
 +
 +
 +
=Solution 4:=
 +
 +
a)
 +
 +
<math>P_\theta(\rho)=F(\rho\text{cos}\theta, \rho\text{sin}\theta)</math>
 +
 +
When <math>f(x,y)=\delta(x,y)</math>, <math>F(\mu,\nu)=1</math>.
 +
 +
So <math>p_\theta(r)=\delta_\theta(r)</math>.
 +
 +
<font color="green"><u>'''Comments'''</u>:
 +
 +
* The answer would be easier to understand if the last statement was expressed in this way: <math>p_{\theta}(r)=\mathcal{F}^{-1}\{ P_{\theta}(\rho)\}=\mathcal{F}^{-1}\{1\}=\delta_{\theta}(r)</math>, where <math>\mathcal{F}^{-1}\{\cdot\}</math> is the Fourier inverse transform.
 +
 +
</font>
 +
 +
 +
b)
 +
 +
When <math>f(x,y)=\delta(x-1,y-1)</math>, <math>F(\mu,\nu)=e^{-j\mu}\cdot e^{-j\nu}</math>.
 +
 +
So <math>P_\theta(\rho)=e^{-j\rho\text{cos}\theta}\cdot e^{-j\rho\text{sin}\theta}</math>, <math>p_\theta(r)=\delta(r-\text{cos}\theta-\text{sin}\theta)</math>.
 +
 +
 +
c)
 +
 +
[[Image:CW_QE_2011_CS5_2_c_sol4.png|200px]]
 +
 +
<math>p_\theta(r)=\int_{-\sqrt{\frac{1}{4}-r^2}}^{\sqrt{\frac{1}{4}-r^2}}1dz=\sqrt{1-4r^2}</math>
 +
 +
So <math>p_\theta(r)=\begin{cases} \sqrt{1-4r^2}, & |r|<\frac{1}{2}\\0, & |r|>\frac{1}{2}\end{cases}</math>.
 +
 +
 +
d)
 +
 +
[[Image:CW_QE_2011_CS5_2_d_sol4.png|300px]]
 +
 +
<math>r_0=\sqrt{2}\cdot\left(\frac{\pi}{4}-\theta\right)</math>
 +
 +
<math>p_\theta(r)=\int_{-\infty}^{\infty}f(r\text{cos}\theta-z\text{sin}\theta, r\text{sin}\theta+z\text{cos}\theta)dz=\int_{-\sqrt{\frac{1}{4}-(r-r_0)^2}}^{\sqrt{\frac{1}{4}-(r-r_0)^2}}adz</math>
 +
 +
So <math>p_\theta(r)=\begin{cases}\sqrt{1-4\left(r-\sqrt{2}\left(\frac{\pi}{4}-\theta\right)\right)}, & |r-r_0|\leq\frac{1}{2}\\0, & \text{otherwise}\end{cases}</math>.
 +
 +
<font color="green"><u>'''Comments'''</u>:
 +
 +
* The computation for <math>r_0</math> is wrong. The correct expression of <math>r_0</math> should be: <math>r_0 = \sqrt{2}\sin(\frac{\pi}{4}-\theta)</math>. It can also be expressed as: <math>r_0=\cos\theta+\sin\theta</math>.
 +
* The rest statement is correct, and then the final result becomes: <math>p_{\theta}(r)=\begin{cases}\sqrt{1-4(r-r_0)}, & |r-r_0|\leq\frac{1}{2}\\0, & \text{otherwise} \end{cases}</math>.
 +
 +
</font>
 +
 +
 +
e)
 +
 +
<math>
 +
\begin{align}
 +
f(x,y) & =\int_{-\infty}^{\infty}\int_{-\infty}^{\infty}F(\mu,\nu)e^{2\pi j(x\mu+y\nu)}d\mu d\nu\\
 +
& = \int_{-\infty}^{\infty}\int_{0}^{\pi}p_\theta(\rho)e^{2\pi j(x\rho\text{cos}\theta+y\rho\text{sin}\theta)}|\rho|d\theta d\rho\\
 +
& = \int_{0}^{\pi}\underbrace{\int_{-\infty}^{\infty}|\rho|p_\theta(\rho)e^{2\pi j(x\rho\text{cos}\theta+y\rho\text{sin}\theta)}d\rho}_{g_\theta(x\text{cos}\theta+y\text{sin}\theta)}d\theta
 +
\end{align}
 
</math>
 
</math>
 +
 +
So <math>g_\theta(t)</math> is given by:
 +
 +
<math>g_\theta(t)=\int_{-\infty}^{\infty}|\rho|p_\theta(\rho)e^{2\pi j\rho t}d\rho=\text{CTFT}^{-1}\{|\rho|p_\theta(\rho)\}=h(t)\ast p_\theta(r)</math>
 +
 +
So what we do is:
 +
 +
# measure projection <math>p_\theta(r)</math>;
 +
# filter the projection <math>g_\theta(r)=h(r)\ast p_\theta(r)</math>;
 +
# back project filtered projection: <math>f(x,y)=\int_0^{\pi}g_\theta(x\text{cos}\theta)+y\text{sin}\theta)d\theta</math>.
 +
 +
 +
----
 +
=Related Problems=
 +
 +
a) Calculate the forward projection <math>p_\theta(r)</math>, for <math>f(x,y)=\text{rect}(x,y)</math>, as <math>\theta=0</math> and <math>\theta=\pi/4</math>.
 +
 +
b) Calculate the forward projection <math>p_\theta(r)</math>, for <math>f(x,y)=\delta(x-a,y-b)</math>.
  
 
----
 
----
Line 258: Line 550:
 
----
 
----
  
[[ECE PhD Qualifying Exams|Back to ECE Qualifying Exams (QE) page]]
+
[[ECE PhD Qualifying Exams|Back to ECE Qualifying Exams (QE) page]]
 
+
[[Category:ECE]] [[Category:QE]] [[Category:Automatic_Control]] [[Category:Problem_solving]]
+

Latest revision as of 01:51, 31 March 2015


ECE Ph.D. Qualifying Exam

Communication, Networking, Signal and Image Processing (CS)

Question 5: Image Processing

August 2011



Part 2

Jump to Part 1,2


 $ \color{blue}\text{Consider an image } f(x,y) \text{ with a forward projection} $

                $ \color{blue} p_{\theta}(r) = \mathcal{FP}\left \{ f(x,y) \right \} $

                             $ \color{blue} = \int_{-\infty}^{\infty}{f \left ( r cos(\theta) - z sin(\theta),r sin(\theta) + z cos(\theta) \right )dz}. $

$ \color{blue} \text{Let } F(\mu,\nu) \text{ be the continuous-time Fourier transform of } f(x,y) \text{ given by} $
              $ \color{blue} F(u,v) = \int_{-\infty}^{\infty}{\int_{-\infty}^{\infty}{f(x,y)e^{-j2\pi(ux,vy)}dx}dy} $

$ \color{blue} \text{and let } P_{\theta}(\rho) \text{ be the continuous-time Fourier transform of } p_{\theta}(r) \text{ given by} $
              $ \color{blue} P_{\theta}(\rho) = \int_{-\infty}^{\infty}{p_{\theta}(r)e^{-j2\pi(\rho r)}dr}. $


$ \color{blue}\text{a) Calculate the forward projection }p_{\theta}(r) \text{, for } f(x,y) = \delta(x,y). $

$ \color{blue}\text{Solution 1:} $

$ p_{\theta}(r) = \int_{-\infty}^{+\infty}{\delta(r cos\theta - z sin\theta, r sin\theta + z cos \theta) dz} $


$ = z \text{ when } \left\{\begin{matrix} r cos \theta - z sin \theta = 0 \\ r sin \theta + z cos \theta = 0 \end{matrix}\right. $


$ = \frac{r cos\theta}{sin \theta}, \theta > 0 $

$ {\color{red} \text{This answer is incorrect. The correct answer is as following:}} $

$ {\color{green} \text{Recall:}} $


$ {\color{green} \text{i) } \int_{-\infty}^{+\infty}{f(g(t)) \delta (t) dt} = f(g(t=0)) \int_{-\infty}^{+\infty}{\delta (t) dt} } $


$ {\color{green} \text{ii) } \int_{-\infty}^{+\infty}{\delta (\alpha t) dt} = \int_{-\infty}^{+\infty}{\delta (u) \frac{du}{|\alpha|}} = \frac{1}{|\alpha|} } $


$ {\color{green} \text{iii) } \delta() \text{ function is separable: } \delta(x,y) = \delta(x) \cdot \delta(y) } $


$ \color{green} \text{ Define } u = r cos\theta - z sin\theta $


$ \color{green} \Rightarrow dz = \frac{du}{|sin\theta|} $


$ \color{green} \text{ Now } $


$ \color{green} p_{\theta}(r) = \int_{-\infty}^{+\infty}{\delta(r cos\theta - z sin\theta, r sin\theta + z cos \theta) dz} $


$ \color{green} p_{\theta}(r) = \int_{-\infty}^{+\infty}{\delta(g(u)) \delta(u) \frac{du}{|sin\theta|}} = \frac{\delta(u=0)}{|sin\theta|} $


$ \color{green} = \frac{\delta(\frac{r}{sin\theta})}{|sin\theta|} = \frac{|sin\theta|}{|sin\theta|} \delta(r) = \delta(r) $


$ \color{blue}\text{Solution 2:} $

.QE 11 CS5 2 a sol2.PNG

$ p_{\theta}(r) = \int_{-\infty}^{+\infty}{\delta(r cos\theta - z sin\theta, r sin\theta + z cos \theta) dz} $


$ = \delta(r) $


$ {\color{green} \text{Here, the student uses the intuitive solution: in this case the answer does not depend on } \theta \text{, since the image just contains a peak at origin. } } $


$ \color{blue}\text{b) Calculate the forward projection }p_{\theta}(r) \text{, for } f(x,y) = \delta(x-1,y-1). $

$ \color{blue}\text{Solution 1:} $

$ p_{\theta}(r) = \int_{-\infty}^{+\infty}{\delta(r cos\theta - z sin\theta - 1, r sin\theta + z cos \theta - 1) dz} $

$ = z \text{ when } \left\{\begin{matrix} r cos \theta - z sin \theta = 1 \\ r sin \theta + z cos \theta = 1 \end{matrix}\right. $


$ = \frac{r cos\theta - 1}{sin \theta}, \theta > 0 $

$ {\color{red} \text{This answer is incorrect. The correct answer is as following:}} $

$ \color{green} \text{ Similar to the solution 1 to part a) we define u: } u = r cos\theta - z sin\theta - 1 $


$ \color{green} \text{ Following the same logic as in part a) we obtain the final answer:} $


$ \color{green} p_{\theta}(r) = \delta(r - (cos\theta + sin \theta)) = \delta(r - \sqrt{2} cos (\theta - \frac{\pi}{4})) $


$ \color{blue}\text{Solution 2:} $

QE 11 CS5 2 b sol2.PNG

$ \tilde{p}_\theta(r) = p_{\theta}(r - \sqrt{1+1} cos(\theta - tan^{-1}(\frac{1}{1}))) $


$ = p_\theta(r - \sqrt{2} cos(\theta - \frac{\pi}{4})) $


$ = \delta(r - \sqrt{2} cos(\theta - \frac{\pi}{4})) $


$ {\color{green} \text{Again, the student uses the intuitive solution: in this case the answer does depend on } \theta \text{, since the peak is shifted from the origin to the point } (1,1). } $



$ \color{blue}\text{c) Calculate the forward projection }p_{\theta}(r) \text{, for } f(x,y) = rect \left(\sqrt[]{x^2+y^2} \right). $


$ \color{blue}\text{Solution 1:} $


$ p_{\theta}(r) = \int_{-\infty}^{+\infty}{rect(\sqrt{(r cos\theta - z sin\theta)^2 + (r sin\theta + z cos \theta)^2)} dz} $


$ \color{green} \text{Recall should be added:} $

$ \color{green} rect(t) = \left\{\begin{matrix} 1, for |t|\leq \frac{1}{2} \\ 0, otherwise \end{matrix}\right. $


$ \color{green} \text{therefore: } $


$ p_{\theta}(r) = \int_{-\sqrt{\frac{1}{4} - r^2}}^{\sqrt{\frac{1}{4} - r^2}}{1 dz} $


$ = \left\{\begin{matrix} \sqrt{1 - 4r^2}, &\text{ if }|r| \leq \frac{1}{2} \\ 0, &\text{ otherwise} \end{matrix}\right. $


$ \color{blue}\text{Solution 2:} $

QE 11 CS5 2 c sol2.PNG

$ p_{\theta}(r) = \int_{-\infty}^{+\infty}{f(r cos\theta - z sin\theta, r sin\theta + z cos \theta) dz} $


$ = \int_{-\sqrt{\frac{1}{4} - r^2}}^{\sqrt{\frac{1}{4} - r^2}}{1 dz} = \sqrt{1 - 4r^2}, \text{ if }|r| \leq \frac{1}{2} $


$ \text{ else } p_{\theta}(r) = 0 $


$ \color{blue}\text{d) Calculate the forward projection }p_{\theta}(r) \text{, for } f(x,y) = rect \left(\sqrt[]{(x-1)^2+(y-1)^2} \right). $

$ \color{blue}\text{Solution 1:} $


$ p_{\theta}(r) = \int_{-\infty}^{+\infty}{rect \left( \sqrt{(r cos\theta - z sin\theta - 1)^2 + (r sin\theta + z cos \theta - 1)^2} \right) dz} $


$ = \int_{-\sqrt{\frac{1}{4} - (r - (cos\theta + sin\theta))^2}}^{\sqrt{\frac{1}{4} - (r - (cos\theta + sin\theta))^2}}{1 dz} $


$ = \left\{\begin{matrix} \sqrt{1 - 4(r - (cos\theta + sin\theta))^2}, &\text{ if }|r| \leq \frac{1}{2} \\ 0, &\text{ otherwise} \end{matrix}\right. $


$ \color{green} \text{ To make it more clear, the following form could be obtained:} $

$ \color{green} = \left\{\begin{matrix} \sqrt{1 - 4(r - \sqrt{2} cos (\theta - \frac{\pi}{4}))^2}, &\text{ if }|r| \leq \frac{1}{2} \\ 0, &\text{ otherwise} \end{matrix}\right. $


$ \color{blue}\text{Solution 2:} $

$ \tilde{p}_\theta(r) = p_{\theta}(r - \sqrt{1+1} cos(\theta - tan^{-1}(\frac{1}{1}))) $


$ = p_\theta(r - \sqrt{2} cos(\theta - \frac{\pi}{4})) $


$ \text{ where } p_\theta(r) = \left\{\begin{matrix} \sqrt{1 - 4r^2}, &\text{ if }|r| \leq \frac{1}{2} \\ 0, &\text{ else} \end{matrix}\right. $


$ {\color{green} \text{Here, the student uses the results from solutions to part b and c.} } $


$ \color{blue}\text{e) Describe in precise detail, the steps required to perform filtered back projection (FBP) reconstruction of } f(x,y). $


$ \color{blue}\text{Solution 1:} $

$ 1. \text{ Compute } \rho_{\theta}(r) $


$ 2. \text{ Compute FT of step 1.} $

$ 3. \text{ Multiply step 2 by the filter } H(\rho) = |\rho| = f_c \left [ rect(\frac{f}{2f_c}) - \Lambda(\frac{f}{f_c}) \right ] $

$ 4. \text{ Compute inverseFT of step 3.} $

$ \color{red} \text{ This answer is not complete. We need the final step:} $


$ \color{red} 5. \text{ Back project } g_{\theta}(r) \text{ (obtained from step 4) and get: } $

$ \color{red} f(x,y) = \int_{0}^{\pi}{g_\theta(xcos\theta + ysin\theta)d\theta} $


$ \color{blue}\text{Solution 2:} $

$ 1. \text{ Measure the projections } \rho_{\theta}(r) \text{ at various angles} $

$ 2. \text{ Filter the projections } \rho_{\theta}(r) \text{ with } h(r) \text{, where } H(\rho) = |\rho| \text{ and get } g_{\theta}(r) $

$ 3. \text{ Back project } g_{\theta}(r) \text{ along } r = xcos\theta + ysin\theta \text{ and get } $

$ f(x,y) = \int_{0}^{\pi}{g_\theta(xcos\theta + ysin\theta)d\theta} $


Solution 3:

a)

$ p_\theta(r)=\begin{cases}1, & \text{if } r=0\\0, & \text{otherwisr}\end{cases}=\delta(r) $.

CW QE 2011 CS5 2 a sol3.png


b)

  • If $ 0\leq\theta<\frac{\pi}{4} $, $ d=\sqrt{2}\text{cos}(\frac{\pi}{4}-\theta)=\sqrt{2}[\text{cos}\frac{\pi}{4}\text{cos}\theta+\text{sin}\frac{\pi}{4}\text{sin}\theta]=\text{cos}\theta+\text{sin}\theta $.

        CW QE 2011 CS5 2 b 1 sol3.png

  • If $ \frac{\pi}{4}\leq \theta < \frac{\pi}{2} $, $ d=\sqrt{2}\text{cos}(\theta-\frac{\pi}{4})=\sqrt{2}\text{cos}(\frac{\pi}{4}-\theta)=\text{cos}\theta+\text{sin}\theta $.

        CW QE 2011 CS5 2 b 2 sol3.png

  • If $ \frac{\pi}{2}\leq \theta < \frac{3\pi}{4} $, $ d=\sqrt{2}\text{sin}(\frac{\pi}{4}-(\theta-\frac{\pi}{2}))=\sqrt{2}\text{sin}(\frac{3\pi}{4}-\theta)=\sqrt{2}[\text{sin}\frac{3\pi}{4}\text{cos}\theta-\text{cos}\frac{3\pi}{4}\text{sin}\theta]=\text{cos}\theta+\text{sin}\theta $.

        CW QE 2011 CS5 2 b 3 sol3.png

  • If $ \frac{3\pi}{4}\leq \theta < 2\pi $, $ d=-\sqrt{2}\text{sin}(\theta-\frac{\pi}{2}-\frac{\pi}{4})=-\sqrt{2}\text{sin}(\theta-\frac{3\pi}{4})=-\sqrt{2}[\text{sin}\theta\text{cos}\frac{3\pi}{4}-\text{cos}\theta\text{sin}\frac{3\pi}{4}]=\text{cos}\theta+\text{sin}\theta $.

        CW QE 2011 CS5 2 b 4 sol3.png

$ \Rightarrow p_\theta(r)=\delta(r-(\text{cos}\theta+\text{sin}\theta)) $ for all $ \theta $.


c)

Since $ f(x,y) $ is symmetric to $ (0,0) $

$ \Rightarrow p_\theta(r)=p_0(r)=\int_{-\sqrt{\frac{1}{4}-r^2}}^{\sqrt{\frac{1}{4}-r^2}}1\cdot dz=2\int_0^{\sqrt{\frac{1}{4}-r^2}}1\cdot dz\text{, for }\vert r\vert<\frac{1}{2} $

CW QE 2011 CS5 2 c sol3.png

$ \Rightarrow p_\theta(r)=2\sqrt{\frac{1}{4}-r^2}\text{rect}(r) $


d)

From part (b), we know that, if $ f(x,y) $ has $ (1,1) $ shift in $ x-y $ plane, the $ p_\theta(r) $ would have $ (\text{cos}\theta+\text{sin}\theta) $ shift in $ r $-direction.

$ \Rightarrow p_\theta(r)=2\sqrt{\frac{1}{4}-(r-(\text{cos}\theta+\text{sin}\theta))^2}\text{rect}(r-(\text{cos}\theta+\text{sin}\theta)) $


e)

  • First, measure the forward projection $ p_\theta(r) $ for sufficient angles $ \theta s $.
  • Second, filter the forward projection with $ h(r): g_\theta(r)=h(r)\ast p_\theta(r) $, where $ h(r)=\text{CTFT}^{-1}\left\{\vert\rho\vert\right\} $.
  • Finally, back project filtered projection to reconstruct $ f(x,y) $. $ f(x,y)=\int_0^\pi g_\theta(x\text{cos}\theta+y\text{sin}\theta)d\theta $.


Solution 4:

a)

$ P_\theta(\rho)=F(\rho\text{cos}\theta, \rho\text{sin}\theta) $

When $ f(x,y)=\delta(x,y) $, $ F(\mu,\nu)=1 $.

So $ p_\theta(r)=\delta_\theta(r) $.

Comments:

  • The answer would be easier to understand if the last statement was expressed in this way: $ p_{\theta}(r)=\mathcal{F}^{-1}\{ P_{\theta}(\rho)\}=\mathcal{F}^{-1}\{1\}=\delta_{\theta}(r) $, where $ \mathcal{F}^{-1}\{\cdot\} $ is the Fourier inverse transform.


b)

When $ f(x,y)=\delta(x-1,y-1) $, $ F(\mu,\nu)=e^{-j\mu}\cdot e^{-j\nu} $.

So $ P_\theta(\rho)=e^{-j\rho\text{cos}\theta}\cdot e^{-j\rho\text{sin}\theta} $, $ p_\theta(r)=\delta(r-\text{cos}\theta-\text{sin}\theta) $.


c)

CW QE 2011 CS5 2 c sol4.png

$ p_\theta(r)=\int_{-\sqrt{\frac{1}{4}-r^2}}^{\sqrt{\frac{1}{4}-r^2}}1dz=\sqrt{1-4r^2} $

So $ p_\theta(r)=\begin{cases} \sqrt{1-4r^2}, & |r|<\frac{1}{2}\\0, & |r|>\frac{1}{2}\end{cases} $.


d)

CW QE 2011 CS5 2 d sol4.png

$ r_0=\sqrt{2}\cdot\left(\frac{\pi}{4}-\theta\right) $

$ p_\theta(r)=\int_{-\infty}^{\infty}f(r\text{cos}\theta-z\text{sin}\theta, r\text{sin}\theta+z\text{cos}\theta)dz=\int_{-\sqrt{\frac{1}{4}-(r-r_0)^2}}^{\sqrt{\frac{1}{4}-(r-r_0)^2}}adz $

So $ p_\theta(r)=\begin{cases}\sqrt{1-4\left(r-\sqrt{2}\left(\frac{\pi}{4}-\theta\right)\right)}, & |r-r_0|\leq\frac{1}{2}\\0, & \text{otherwise}\end{cases} $.

Comments:

  • The computation for $ r_0 $ is wrong. The correct expression of $ r_0 $ should be: $ r_0 = \sqrt{2}\sin(\frac{\pi}{4}-\theta) $. It can also be expressed as: $ r_0=\cos\theta+\sin\theta $.
  • The rest statement is correct, and then the final result becomes: $ p_{\theta}(r)=\begin{cases}\sqrt{1-4(r-r_0)}, & |r-r_0|\leq\frac{1}{2}\\0, & \text{otherwise} \end{cases} $.


e)

$ \begin{align} f(x,y) & =\int_{-\infty}^{\infty}\int_{-\infty}^{\infty}F(\mu,\nu)e^{2\pi j(x\mu+y\nu)}d\mu d\nu\\ & = \int_{-\infty}^{\infty}\int_{0}^{\pi}p_\theta(\rho)e^{2\pi j(x\rho\text{cos}\theta+y\rho\text{sin}\theta)}|\rho|d\theta d\rho\\ & = \int_{0}^{\pi}\underbrace{\int_{-\infty}^{\infty}|\rho|p_\theta(\rho)e^{2\pi j(x\rho\text{cos}\theta+y\rho\text{sin}\theta)}d\rho}_{g_\theta(x\text{cos}\theta+y\text{sin}\theta)}d\theta \end{align} $

So $ g_\theta(t) $ is given by:

$ g_\theta(t)=\int_{-\infty}^{\infty}|\rho|p_\theta(\rho)e^{2\pi j\rho t}d\rho=\text{CTFT}^{-1}\{|\rho|p_\theta(\rho)\}=h(t)\ast p_\theta(r) $

So what we do is:

  1. measure projection $ p_\theta(r) $;
  2. filter the projection $ g_\theta(r)=h(r)\ast p_\theta(r) $;
  3. back project filtered projection: $ f(x,y)=\int_0^{\pi}g_\theta(x\text{cos}\theta)+y\text{sin}\theta)d\theta $.



Related Problems

a) Calculate the forward projection $ p_\theta(r) $, for $ f(x,y)=\text{rect}(x,y) $, as $ \theta=0 $ and $ \theta=\pi/4 $.

b) Calculate the forward projection $ p_\theta(r) $, for $ f(x,y)=\delta(x-a,y-b) $.


"Communication, Networks, Signal, and Image Processing" (CS)- Question 5, August 2011

Go to


Back to ECE Qualifying Exams (QE) page

Alumni Liaison

Ph.D. 2007, working on developing cool imaging technologies for digital cameras, camera phones, and video surveillance cameras.

Buyue Zhang